Linear Algebra- Linear Transformations

Click For Summary
The discussion revolves around solving a problem related to a linear transformation T: R3 --> R4, with given outputs for specific inputs. Participants express confusion about applying linearity conditions to find T(-8, 1, -3). One user suggests using the linearity property T(ru + sv) = rT(u) + sT(v) to derive a potential solution. After some trial and error, they conclude that setting T(...) equal to T(-8, 1, -3) is the key to finding the correct answer. The conversation highlights the challenges of understanding linear transformations and emphasizes the importance of leveraging known properties for problem-solving.
FinalStand
Messages
60
Reaction score
0

Homework Statement


Let T: R3--> R4 be a linear transformation. Assume that T(1,-2,3) = (1,2,3,4), T(2,1,-1)=(1,0,-1,0)
Which of the following is T(-8,1-3)?

A. (-5,-4,-3,-8)
B. (-5,-4,-3,8)
C. (-5,-4,3,-8).
D.(-5,4,3,-8)
E (-5,4,-3,8)
F. None of the above.

Homework Equations



I really have no clue how to solve this problem. All I know is how to verify if it is linear transformation or not by verifying T(u+v) = T(u) + T(v) and cT(u) = T(c(u)).
And that T(x) = Ax and also that T(0)=0

The Attempt at a Solution



I tried applying those above but nothing works for me. To be honest I have no clue how to start, I tried to find patterns but failed to identify any. Any hints?

Thanks
 
Last edited by a moderator:
Physics news on Phys.org
You can combine those two conditions for linearity and say ##T(r\vec{u}+s\vec{v}) = rT(\vec{u}) + sT(\vec{v})##. Try think how that might be useful in light of the information you've been given about T.
 
I got something like T(r+2s, -2r+s, 3r-s) = (r+s, 2r, 3r-s, 4r) Then the answer could be A. But how do you actually do the question how tdo you solve that? I just did by guessing and inputing numbers to get one of the choices (A-F).
 
Oh I got it, you just have to set the T(...) = T(-8,1,-3) :D
 
Yup. :smile:
 
Question: A clock's minute hand has length 4 and its hour hand has length 3. What is the distance between the tips at the moment when it is increasing most rapidly?(Putnam Exam Question) Answer: Making assumption that both the hands moves at constant angular velocities, the answer is ## \sqrt{7} .## But don't you think this assumption is somewhat doubtful and wrong?

Similar threads

  • · Replies 4 ·
Replies
4
Views
3K
  • · Replies 2 ·
Replies
2
Views
1K
  • · Replies 2 ·
Replies
2
Views
1K
  • · Replies 3 ·
Replies
3
Views
2K
  • · Replies 1 ·
Replies
1
Views
1K
  • · Replies 18 ·
Replies
18
Views
2K
  • · Replies 4 ·
Replies
4
Views
2K
Replies
8
Views
2K
  • · Replies 26 ·
Replies
26
Views
3K
  • · Replies 11 ·
Replies
11
Views
2K